Arc Length of Parametric Curve: t = 0 to t = 1

Click For Summary
The discussion revolves around finding the arc length of a parametric curve defined by x = 4t^2 and y = 2t, over the interval from t = 0 to t = 1. The correct formula for parametric arc length is confirmed, and the user calculates derivatives dx/dt and dy/dt, leading to an integral involving a trigonometric substitution. However, several participants point out errors in the integration process, particularly in the handling of integrals and substitutions. The conversation emphasizes the complexity of the problem, with suggestions for alternative approaches and clarifications on the integration steps needed to arrive at the correct solution. Understanding the integration process is deemed more important than simply obtaining the right answer.
sassafrasaxe
Messages
12
Reaction score
0

Homework Statement



Find the arc length of a curve given parametrically from t = 0 to t = 1.
Curve given by x = 4t^2, y = 2t


Homework Equations



[I think] parametric arclength =
integral from t = b to t = a of sqrt( (dx/dt)^2 + (dy/dt)^2)dt


The Attempt at a Solution



dx/dt = 8t, and dy/dt = 2
Then I did S from 0 to 1 of sqrt((8t)^2) + (2)^2)dt

I then drew a triangle for trigonometric substitution, with the tangent case.
Let t = (1/4)tan(σ)
Let dt = (1/4)sec^2(σ) dσ
Let sqrt(64t^2 + 4) = 4sec(σ)
Then S[ sqrt( (8t)^2 + (2)^2) ]dt becomes
S of 4sec(σ) * (1/4)sec^2(σ) dσ
which equals S[ sec^3(σ) ] dσ

Now I solved that with integration by parts, letting u = sec(σ), and dv = sec^2(σ)
so that du = sec(σ)tan(σ)dσ and v = tan(σ)
now S[ sec^3(σ) ]dσ = sec(σ)tan(σ) - S[ tan^3(σ)sec(σ) ]dσ

that last little integral = S[ tan^3(σ)sec(σ) ]dσ = S[ ((sec^2(σ) - 1)*sec(σ)tan(σ)]dσ]
=(1/3)sec^3(σ) - sec(σ)
so the entire S[ sec^3(σ) ]dσ = sec(σ)tan(σ) - (1/3)sec^3(σ) + sec(σ)
so (4/3)S = sec(σ) + sec(σ)*tan(σ)
so S = (3/4)[sec(σ)tan(σ) + sec(σ)]

Now I'm replacing this value with the value given by my triangle for sec(σ) and tan(σ)
so that S = (3/4)[sqrt(64t^2 + 4)*4t + sqrt(64t^2 + 4)] evaluated from t = 0 to t = 1
which would be
(3)sqrt(68) + (3/4)sqrt(68)
minus
(6/4) + 2
=
(15/4)sqrt(68) - (14/4) as my final answer.

Could someone please tell me if I'm doing anything wrong? Maybe if I'm going about this entirely the wrong way? Unfortunately I don't even know the right answer, but even if I can't get the right answer, as long as I understand how to get there, I feel okay.

I would really appreciate it, and I'm sorry about the typing, I'm not very good at typing these problems out. So much thanks,
Sassy
 
Physics news on Phys.org
sassafrasaxe said:

Homework Statement



Find the arc length of a curve given parametrically from t = 0 to t = 1.
Curve given by x = 4t^2, y = 2t


Homework Equations



[I think] parametric arclength =
integral from t = b to t = a of sqrt( (dx/dt)^2 + (dy/dt)^2)dt
Yes, that's correct.


The Attempt at a Solution



dx/dt = 8t, and dy/dt = 2
Then I did S from 0 to 1 of sqrt((8t)^2) + (2)^2)dt

I then drew a triangle for trigonometric substitution, with the tangent case.
Let t = (1/4)tan(σ)

Let dt = (1/4)sec^2(σ) dσ[/quote]
No big deal but I wouldn't use the word "let" here. You already said "let t= (1/4)tan(σ) and then dt= (1/4)sec^2(σ) follows from that.

Let sqrt(64t^2 + 4) = 4sec(σ)
Same point

Then S[ sqrt( (8t)^2 + (2)^2) ]dt becomes
S of 4sec(σ) * (1/4)sec^2(σ) dσ
which equals S[ sec^3(σ) ] dσ

Now I solved that with integration by parts, letting u = sec(σ), and dv = sec^2(σ)
so that du = sec(σ)tan(σ)dσ and v = tan(σ)
now S[ sec^3(σ) ]dσ = sec(σ)tan(σ) - S[ tan^3(σ)sec(σ) ]dσ
How did you get this? with du= sec(\sigma)tan(\sigma)d\sigma)dxand v= tan(\sigma), as you say, that last integrand is sec(\sigma)tan^2(\sigma)d\sigma, not tan^3(\sigma)sec(\sigma)d\sigma.<br /> <br /> <blockquote data-attributes="" data-quote="" data-source="" class="bbCodeBlock bbCodeBlock--expandable bbCodeBlock--quote js-expandWatch"> <div class="bbCodeBlock-content"> <div class="bbCodeBlock-expandContent js-expandContent "> that last little integral = S[ tan^3(σ)sec(σ) ]dσ = S[ ((sec^2(σ) - 1)*sec(σ)tan(σ)]dσ]<br /> =(1/3)sec^3(σ) - sec(σ)<br /> so the entire S[ sec^3(σ) ]dσ = sec(σ)tan(σ) - (1/3)sec^3(σ) + sec(σ)<br /> so (4/3)S = sec(σ) + sec(σ)*tan(σ)<br /> so S = (3/4)[sec(σ)tan(σ) + sec(σ)] </div> </div> </blockquote> With tan^2(\sigma) sec(\sigma)d\sigma, I would proceed by putting everything in terms of sine and cosine:<br /> \int tan^2(\sigma) sec(\sigma)d\sigma= \int \frac{sin^2(\sigma)}{cos^2(\sigma)}\frac{1}{cos(\sigma)}d\sigma= \int \frac{sin^2(\sigma)}{cos^3(\sigma)}d\sigma<br /> <br /> With that odd power of cosine, I would multiply both numerator and denominator bycos(\sigma) to get
\int \frac{sin^2(\sigma)}{cos^4(\sigma)}cos(\sigma)d\sigma= \int\frac{sin^2(\sigma)}{(1- sin^2(\sigma))^2} cos(\sigma)d\sigma
and use the substitution u= sin(\sigma), du= cos(\sigma)d\sigma so that the integral becomes
\int \frac{u^2}{(1- u^2)^2}du

Now I'm replacing this value with the value given by my triangle for sec(σ) and tan(σ)
so that S = (3/4)[sqrt(64t^2 + 4)*4t + sqrt(64t^2 + 4)] evaluated from t = 0 to t = 1
which would be
(3)sqrt(68) + (3/4)sqrt(68)
minus
(6/4) + 2
=
(15/4)sqrt(68) - (14/4) as my final answer.

Could someone please tell me if I'm doing anything wrong? Maybe if I'm going about this entirely the wrong way? Unfortunately I don't even know the right answer, but even if I can't get the right answer, as long as I understand how to get there, I feel okay.

I would really appreciate it, and I'm sorry about the typing, I'm not very good at typing these problems out. So much thanks,
Sassy
 
  • Like
Likes 1 person
Let sqrt(64t^2 + 4) = 4sec(σ)

Should this be 2sec(σ)?

Then also, let t = 1/4 sinh(σ) for a shorter solution, although this is an example of why I disliked integration, it is sometimes very arcane.
 
Okay great thank you very much. So I took it from where you left off, HallsofIvy, and I see what you mean that yes, I did mess up with the integral and it should have been tan^2(σ)sec(σ) d(σ).
Picking up where you left off, I suppose I would then need partial fractions?
Wow, this has been quite the problem! Looks deceivingly simple.
 
Question: A clock's minute hand has length 4 and its hour hand has length 3. What is the distance between the tips at the moment when it is increasing most rapidly?(Putnam Exam Question) Answer: Making assumption that both the hands moves at constant angular velocities, the answer is ## \sqrt{7} .## But don't you think this assumption is somewhat doubtful and wrong?

Similar threads

  • · Replies 2 ·
Replies
2
Views
1K
  • · Replies 3 ·
Replies
3
Views
2K
  • · Replies 2 ·
Replies
2
Views
1K
  • · Replies 6 ·
Replies
6
Views
2K
Replies
7
Views
2K
  • · Replies 3 ·
Replies
3
Views
3K
  • · Replies 22 ·
Replies
22
Views
3K
  • · Replies 10 ·
Replies
10
Views
1K
Replies
6
Views
1K
Replies
2
Views
2K